Những câu hỏi liên quan
Tuyển Trần Thị
Xem chi tiết
alibaba nguyễn
19 tháng 10 2017 lúc 16:10

Xem lại cái đề đi Tuyển. Hình như giá trị nhỏ nhất của cái biểu thức dưới còn lớn hơn là 1 thì làm sao bài đó có giá trị x, y, z thỏa được mà bảo tính A.

Nguyễn Minh Hiển
Xem chi tiết
Trần Thị Mỹ Duyên
Xem chi tiết
mr. killer
Xem chi tiết
Lyzimi
Xem chi tiết
Thắng Nguyễn
27 tháng 8 2017 lúc 9:35

Từ \(xy+yz+xz=xyz\Rightarrow\frac{1}{x}+\frac{1}{y}+\frac{1}{z}=1\)

Đặt \(\left(\frac{1}{x};\frac{1}{y};\frac{1}{z}\right)\rightarrow\left(a,b,c\right)\) thì có

\(\frac{c^3}{\left(a+1\right)\left(b+1\right)}+\frac{b^3}{\left(a+1\right)\left(c+1\right)}+\frac{a^3}{\left(b+1\right)\left(c+1\right)}\ge\frac{1}{16}\)\(\forall\hept{\begin{cases}a+b+c=1\\a,b,c>0\end{cases}}\)

Áp dụng BĐT AM-GM ta có:

\(\frac{a^3}{\left(b+1\right)\left(c+1\right)}+\frac{b+1}{64}+\frac{c+1}{64}\ge\frac{3a}{16}\)

Tương tự cho 2 BĐT còn lại rồi cộng theo vế

\(VT+\frac{2\left(a+b+c+3\right)}{64}\ge\frac{3\left(a+b+c\right)}{16}\Leftrightarrow VT\ge\frac{1}{16}\)

Khi \(a=b=c=\frac{1}{3}\Leftrightarrow x=y=z=1\)

BaBie
24 tháng 8 2017 lúc 15:12

a) x4+x3+2x2+x+1=(x4+x3+x2)+(x2+x+1)=x2(x2+x+1)+(x2+x+1)=(x2+x+1)(x2+1)

b)a3+b3+c3-3abc=a3+3ab(a+b)+b3+c3 -(3ab(a+b)+3abc)=(a+b)3+c3-3ab(a+b+c)

=(a+b+c)((a+b)2-(a+b)c+c2)-3ab(a+b+c)=(a+b+c)(a2+2ab+b2-ac-ab+c2-3ab)=(a+b+c)(a2+b2+c2-ab-ac-bc)

c)Đặt x-y=a;y-z=b;z-x=c

a+b+c=x-y-z+z-x=o

đưa về như bài b

d)nhóm 2 hạng tử đầu lại và 2hangj tử sau lại để 2 hạng tử sau ở trong ngoặc sau đó áp dụng hằng đẳng thức dề tính sau đó dặt nhân tử chung

e)x2(y-z)+y2(z-x)+z2(x-y)=x2(y-z)-y2((y-z)+(x-y))+z2(x-y)

=x2(y-z)-y2(y-z)-y2(x-y)+z2(x-y)=(y-z)(x2-y2)-(x-y)(y2-z2)=(y-z)(x2-2y2+xy+xz+yz)

Lyzimi
24 tháng 8 2017 lúc 16:10

BaBie làm cái chi đây 

Xem chi tiết
Nguyễn Linh Chi
5 tháng 7 2019 lúc 11:49

Xét tích : \(\left[x^2\left(z-y\right)+y^2\left(x-z\right)+z^2\left(y-x\right)\right]\left(x+y+z\right)\)

=\(x^3\left(z-y\right)+x^2\left(z-y\right)\left(z+y\right)+y^3\left(x-z\right)+y^2\left(x-z\right)\left(x+z\right)\)

\(+z^3\left(y-x\right)+z^2\left(y-x\right)\left(y+x\right)\)

\(=x^3\left(z-y\right)+y^3\left(x-z\right)+z^3\left(y-x\right)+x^2\left(z^2-y^2\right)+y^2\left(x^2-z^2\right)+z^2\left(y^2-x^2\right)\)

\(=x^3\left(z-y\right)+y^3\left(x-z\right)+z^3\left(y-x\right)+x^2z^2-x^2y^2+y^2x^2-y^2z^2+z^2y^2-z^2x^2\)

\(=x^3\left(z-y\right)+y^3\left(x-z\right)+z^3\left(y-x\right)\)

Như vậy:

 \(\left[x^2\left(z-y\right)+y^2\left(x-z\right)+z^2\left(y-x\right)\right]\left(x+y+z\right)\)\(=x^3\left(z-y\right)+y^3\left(x-z\right)+z^3\left(y-x\right)\)

<=> \(\frac{x^3\left(z-y\right)+y^3\left(x-z\right)+z^3\left(y-x\right)}{x^2\left(z-y\right)+y^2\left(x-z\right)+z^2\left(y-x\right)}=x+y+z\)

Ta có: \(\frac{\frac{x^2\left(z-y\right)}{yz}+\frac{y^2\left(x-z\right)}{xz}+\frac{z^2\left(y-x\right)}{xy}}{\frac{x\left(z-y\right)}{yz}+\frac{y\left(x-z\right)}{xz}+\frac{z\left(y-x\right)}{xy}}\)

 \(=\frac{\frac{x^3\left(z-y\right)}{xyz}+\frac{y^3\left(x-z\right)}{xyz}+\frac{z^3\left(y-x\right)}{xyz}}{\frac{x^2\left(z-y\right)}{xyz}+\frac{y^2\left(x-z\right)}{xyz}+\frac{z^2\left(y-x\right)}{xyz}}\)

\(=\frac{x^3\left(z-y\right)+y^3\left(x-z\right)+z^3\left(y-x\right)}{x^2\left(z-y\right)+y^2\left(x-z\right)+z^2\left(y-x\right)}=x+y+z\)

nguyễn phương thảo
Xem chi tiết
Hoàng Lê Bảo Ngọc
8 tháng 8 2016 lúc 17:33

Bạn xem lại đề nhé :)

Thay 1 bằng xy + yz + zx được : 

\(1+y^2=xy+yz+zx+y^2=x\left(y+z\right)+y\left(y+z\right)=\left(x+y\right)\left(y+z\right)\)

Tương tự : \(1+x^2=\left(x+y\right)\left(x+z\right)\)\(1+z^2=\left(x+z\right)\left(z+y\right)\)

Suy ra \(Q=x\sqrt{\frac{\left(x+y\right)\left(y+z\right).\left(x+z\right)\left(z+y\right)}{\left(x+y\right)\left(x+z\right)}}+y\sqrt{\frac{\left(x+y\right)\left(x+z\right).\left(z+x\right)\left(z+y\right)}{\left(x+y\right)\left(y+z\right)}}+z\sqrt{\frac{\left(x+y\right)\left(x+z\right).\left(x+y\right)\left(y+z\right)}{\left(x+z\right)\left(z+y\right)}}\)

\(=x\sqrt{\left(y+z\right)^2}+y\sqrt{\left(x+z\right)^2}+z\sqrt{\left(x+y\right)^2}=x\left|y+z\right|+y\left|x+z\right|+z\left|x+y\right|\)

\(=2\left(xy+yz+zx\right)=2\)(vì x,y,z > 0)

Đà Giang
Xem chi tiết
alibaba nguyễn
28 tháng 9 2018 lúc 13:50

\(xy+yz+zx=xyz\)

\(\Leftrightarrow\frac{1}{x}+\frac{1}{y}+\frac{1}{z}=1\)

Đặt \(\frac{1}{x}=a;\frac{1}{y}=b;\frac{1}{z}=c\) thì

\(\hept{\begin{cases}a+b+c=1\\P=\frac{a^3}{\left(1+b\right)\left(1+c\right)}+\frac{b^3}{\left(1+c\right)\left(1+a\right)}+\frac{c^3}{\left(1+a\right)\left(1+b\right)}\ge\frac{1}{16}\end{cases}}\)

Ta co:

\(\frac{a^3}{\left(1+b\right)\left(1+c\right)}+\frac{1+b}{64}+\frac{1+c}{64}\ge\frac{3a}{16}\)

\(\Leftrightarrow\frac{a^3}{\left(1+b\right)\left(1+c\right)}\ge\frac{3a}{16}-\frac{b}{64}-\frac{c}{64}-\frac{1}{32}\)

Từ đây ta co:

\(P\ge\left(a+b+c\right)\left(\frac{3}{16}-\frac{1}{64}-\frac{1}{64}\right)-\frac{3}{32}=\frac{1}{16}\)

MInemy Nguyễn
Xem chi tiết